上一主题:Schweser Online Exam
下一主题:CFA Sample I
返回列表 发帖

Market integrity

When the ask and bid is too wide, can I place an order opposite of what I intend to narrow the spread? For example, ask is 40, bid 42. I want to buy. What if I first place an order to sell at 41.5 hoping mm will front run my order and then I buy at 41.50 or less?

Does this violate any rule?

no, this is perfectly fine. if the bid were 40, ask 42, you could place an order to sell at 41.5 and the bid/offer would go to 41.5 x 42 (assuming you have a round lot order). however, a market maker in receiving your order couldn't print an order for itself at 41.75 and then print you at 41.5. your order if it's inside bid/ask takes precedence over the market maker order. in NASD they created a rule to protect customer orders called the manning rule that is pretty much just this... a market maker can't front run a client order. to tighten up a spread (but not necessarily print anything to the tape), either a client or a market maker could post up an order to narrow the spread. they then become the inside market.

so in your case, you narrow the spread to 41.5 x 42... you want to buy, but chances are if anything you may get your order off on the sell side. granted, if you wanted to buy 20k shrs and you placed a limit for 100 shares to sell in this day and age where a lot of orders will follow bids/offers, who knows... maybe you could do yourself a favor, but you probably would be better off placing the buy order at 41.5 or 41 limit and see if your order gets filled than go other side of what you're intending to do.

TOP

Thanks bannisja. You are right bid is 40 and ask is 42.

In my experience, when I put in a sell order of 41.50, very likely mm will match that (I can tell by the sell size). Also If I cancel my order, the ask will likely go back to 42 immediately. So what do you call this? What is the motivation of mm?

If someone hits the ask of 41.5, whose order will be filled? If it is my order, why mm match up my order at the first place?

What if I withdraw the 41.5 sell order after seeing 41.5 ask will stay and then immediately buy at 41.5? I.e., my motivation is clear - lower the ask with no real intention to sell, is this still ok?

I do not want to post inside limit buy order because I fear the ask will be raised.

TOP

so the MM could match you at 41.5- say you have 100 shrs and they want to make a sort of efficient market, they could post bigger size. Also, a lot of electronic orders these days are such that you can put in a limit order that will follow the inside market for (blah) amount of shares to a limit. Sort of like when you go on Ebay and say you'll chase an auction price up to (blah), but don't want to monitor it all the time or just show your cards and top price right away. your order goes in for 100 shrs and yes, you may suddenly see 1k or so on the lot size inside quoted. if yours is the first order, the responsibility then is to fill your order first if a print goes off, before a market maker can take an order for themself. so if inside mkt now is 40 x 41.5 b/c of your order and someone comes in with a market order to buy (you had placed a sell), then you'll get filled at 41.5 as your order is 1st in line. there are priorities of orders based on time, size, etc, but a mkt maker cannot front run you.

if you withdraw your order, if an automated ECN, you'll see the inside market go back to 40x42 almost instantaneously. if it's a market maker actually working your order, they have something like 30 or 60 seconds to pull away that order, so if you placed a mkt order after you cxl to buy but the mkt maker had not updated their quote, so long as the timing makes sense and they update their market in the time, you could go unfilled, see the 41.5 go away on the buy side, and them post you 41.5 x 42 bid/ask as they update their quote. that is not illegal. if they printed stock, though, you'd get filled... again, no front running your order either side of the mkt.

people are allowed to place orders and not have them shown. if an order has a qualifier also like an ALL OR NONE, the order does not get shown b/c of the stipulation on it. if a quote were that wide, i would place a limit in between for sure. if the market has a 1 or 2 cent spread and is a liquid market, then probably playing games like that isn't really worth it. market making used to be a huge $$$$ business. with decimalization and rules like the manning rule, etc, it's very hard now a days to make a ton of money doing it. efficiencyy has improved dramatically and the clients placing orders have been the beneficiaries.

TOP

返回列表
上一主题:Schweser Online Exam
下一主题:CFA Sample I